As it is presented in the passage, the approach to history taken by mainstream U.S. historians of the late nineteenth...

Jasherrell on February 4, 2018

Example 2 Question 3

Hello, I do not understand why A would be the answer and not C. Since the rule is if Mushrooms are 3rd then Lentils are last and if Mushrooms arent 3rd, lentils cannot be last.

Reply
Create a free account to read and take part in forum discussions.

Already have an account? log in

Mehran on February 9, 2018

The rule you are referring to is:

M3 ==> L6
Not L6 ==> not M3

Here we are told that the lentils are added last, i.e. L6, but that is our necessary condition.

The existence of the necessary condition tells us nothing.

You are making an incorrect reversal by assuming it must be true that the mushrooms are added third.

Hope that helps! Please let us know if you have any other questions.